How Does the Dirac Equation Utilize Component Functions in Its Derivation?

In summary, the solution to the Dirac equation is obtained by extending the equation to four components.
  • #1
exponent137
561
33
In
https://quantummechanics.ucsd.edu/ph130a/130_notes/node45.html
after
"Instead of an equation which is second order in the time derivative, we can make a first order equation, like the Schrödinger equation, by extending this equation to four components."

it is evident that the solution is obtained with help of ##a^2-b^2=(a+b)(a-b)##

I cannot follow in this derivation, how rows ##\phi^{(L)}=...## and ##\phi^{(R)}=...## are used. Maybe more steps instead of these two rows will help.

Although I think that Feynman once described this more clearly in his book about QED.

Can someone, please, gives a link or more clearly explains this type of derivation of Dirac equation?
 
Last edited:
Physics news on Phys.org
  • #2
I'm not sure what your point of confusion is. Do you understand that the following equations are true, by definition of ##\phi^{(R)}## and ##\phi^{(L)}## (plus the fact that ##\phi## obeys the second-order equation)?

##(i \hbar \frac{\partial}{\partial t} - i \hbar \sigma \cdot \nabla) \phi^{(R)} = mc \phi^{(L)}##

##(i \hbar \frac{\partial}{\partial t} + i \hbar \sigma \cdot \nabla)\phi^{(L)} = mc \phi^{(R)}##
 
  • Like
Likes exponent137
  • #3
stevendaryl said:
I'm not sure what your point of confusion is. Do you understand that the following equations are true, by definition of ##\phi^{(R)}## and ##\phi^{(L)}## (plus the fact that ##\phi## obeys the second-order equation)?

##(i \hbar \frac{\partial}{\partial t} - i \hbar \sigma \cdot \nabla) \phi^{(R)} = mc \phi^{(L)}##

##(i \hbar \frac{\partial}{\partial t} + i \hbar \sigma \cdot \nabla)\phi^{(L)} = mc \phi^{(R)}##

If I put the second equation in the first one, I obtain:

##(i \hbar \frac{\partial}{\partial t} - i \hbar \sigma \cdot \nabla)(i \hbar \frac{\partial}{\partial t}+ i \hbar \sigma \cdot \nabla)\phi^{(L)} = (mc)^2 \phi^{(L)}##

One problem is solved, I think that now I understand this derivation of these two rows, as I mentioned. Thanks.

I think that Feynman used this type of calculation, as you wrote. But I think that he continued I little bit more cleary that in my link?
 

Related to How Does the Dirac Equation Utilize Component Functions in Its Derivation?

1. What is the Dirac equation?

The Dirac equation is a mathematical equation that describes the behavior of fermions, which are particles with half-integer spin. It was formulated by physicist Paul Dirac in 1928 and is a fundamental equation in quantum mechanics.

2. What does the Dirac equation describe?

The Dirac equation describes the behavior of particles such as electrons and quarks, which are considered elementary particles in the Standard Model of particle physics. It takes into account both special relativity and quantum mechanics, making it a crucial tool for understanding the behavior of these particles.

3. How is the Dirac equation different from the Schrödinger equation?

The Schrödinger equation only applies to particles with integer spin, while the Dirac equation applies to both particles with integer and half-integer spin. Additionally, the Dirac equation takes into account the effects of special relativity, while the Schrödinger equation does not.

4. What is the significance of the Dirac equation?

The Dirac equation has had a profound impact on our understanding of the subatomic world. It helped reconcile special relativity with quantum mechanics and has been crucial in predicting and explaining the behavior of particles in high-energy physics experiments.

5. How is the Dirac equation used in modern physics?

The Dirac equation is used in a variety of fields, including particle physics, quantum field theory, and condensed matter physics. It has also been applied in the development of technologies such as transistors and lasers. Additionally, the Dirac equation has been used in the study of exotic particles and phenomena, such as antimatter and black holes.

Similar threads

  • Quantum Physics
Replies
1
Views
919
  • Quantum Physics
Replies
2
Views
1K
  • Quantum Physics
2
Replies
40
Views
3K
  • Quantum Physics
Replies
13
Views
2K
Replies
14
Views
1K
  • High Energy, Nuclear, Particle Physics
Replies
1
Views
1K
Replies
4
Views
499
  • Quantum Physics
Replies
3
Views
3K
Replies
4
Views
2K
Back
Top